Finding inverse of a Laplace transform by convolution

Click For Summary
The discussion revolves around finding the inverse Laplace transform of the function F(s) = s/((s+1)(s^2+4)) using the convolution theorem. The correct approach involves recognizing that the convolution integral should be a definite integral and should include cos(2τ) instead of cos(τ). The participant attempts to solve the problem using partial fractions, yielding a different result for the inverse transform. Clarifications emphasize the importance of limits in convolution and suggest verifying the results through differentiation. The conversation highlights common pitfalls in applying the convolution theorem correctly.
Pouyan
Messages
103
Reaction score
8

Homework Statement


find the inverse Laplace transform of the given function by
using the convolution theorem

Homework Equations



F(s) = s/((s+1)(s2)+4)

The theorem : Lap{(f*g)(t)} = F(s)*G(s)

The Attempt at a Solution


I know how to find it the answer is :
we have 1/(s+1) * s/(s+4) and the inverse of each of these functions are : e-t * cos(2t)
further the answer is : ∫(e(-(t-τ))*cos(τ)dτ)
But if I try to solve this problem without convolution theorem; and with partial fraction I get :

s/((s+1)(s2+4)) = (1/5) ( (1/(s+1) + s/(s2+4) + 4/(s2+4) )

and the inverse of this function is :

(1/5) (cos(2t) - e-t +2sin(2t))

MY QUESTION IS :

∫(e(-(t-τ))*cos(τ)dτ) = (1/5) (cos(2t) - e-t +2sin(2t)) is this right ?
 
Physics news on Phys.org
Pouyan said:

Homework Statement


find the inverse Laplace transform of the given function by
using the convolution theorem

Homework Equations



F(s) = s/((s+1)(s2)+4)

The theorem : Lap{(f*g)(t)} = F(s)*G(s)

The Attempt at a Solution


I know how to find it the answer is :
we have 1/(s+1) * s/(s+4) and the inverse of each of these functions are : e-t * cos(2t)
further the answer is : ∫(e(-(t-τ))*cos(τ)dτ)
But if I try to solve this problem without convolution theorem; and with partial fraction I get :

s/((s+1)(s2+4)) = (1/5) ( (1/(s+1) + s/(s2+4) + 4/(s2+4) )

and the inverse of this function is :

(1/5) (cos(2t) - e-t +2sin(2t))

MY QUESTION IS :

∫(e(-(t-τ))*cos(τ)dτ) = (1/5) (cos(2t) - e-t +2sin(2t)) is this right ?

What you wrote is not the convolution; the convolution is a definite integral, and you wrote an indefinite integral. In this method especially, limits are crucial.

Anyway, in your integral you should have ##\cos(2 \tau) \, d \tau##, not ##\cos(\tau) \, d \tau##.

After fixing things up, you will be able to answer your own question, by either (i) doing the integral; or (ii) differentiating both sides to see if the derivatives match.
 
Question: A clock's minute hand has length 4 and its hour hand has length 3. What is the distance between the tips at the moment when it is increasing most rapidly?(Putnam Exam Question) Answer: Making assumption that both the hands moves at constant angular velocities, the answer is ## \sqrt{7} .## But don't you think this assumption is somewhat doubtful and wrong?

Similar threads

  • · Replies 1 ·
Replies
1
Views
1K
  • · Replies 10 ·
Replies
10
Views
2K
Replies
2
Views
2K
  • · Replies 4 ·
Replies
4
Views
3K
Replies
3
Views
2K
  • · Replies 12 ·
Replies
12
Views
4K
  • · Replies 2 ·
Replies
2
Views
1K
  • · Replies 2 ·
Replies
2
Views
3K
Replies
2
Views
2K
  • · Replies 3 ·
Replies
3
Views
2K